2004 AMC 12A Problems/Problem 5

Revision as of 08:44, 15 April 2008 by 1=2 (talk | contribs) (New page: ==Problem== The graph of the line <math>y=mx+b</math> is shown. Which of the following is true? Image:2004 AMC 12A Problem 5.png <math>\mathrm {(A)} mb<-1 \qquad \mathrm {(B)} -1<mb<...)
(diff) ← Older revision | Latest revision (diff) | Newer revision → (diff)

Problem

The graph of the line $y=mx+b$ is shown. Which of the following is true?

2004 AMC 12A Problem 5.png

$\mathrm {(A)} mb<-1 \qquad \mathrm {(B)} -1<mb<0 \qquad \mathrm {(C)} mb=0 \qquad \mathrm {(D)} 0<mb<1 \qquad \mathrm {(E)} mb>1$

Solution

It looks like it has a slope of $-\dfrac{1}{2}$ and is shifted $\dfrac{4}{5}$ up. $\dfrac{4}{5}\cdot \dfrac{-1}{2}=\dfrac{-4}{10} \Rightarrow \mathrm {(B)}$

See also